L'Hôpital's Rule | Brilliant Math & Science Wiki (2024)

Sign up with Facebook or Sign up manually

Already have an account? Log in here.

Chungsu Hong, Pi Han Goh, Thaddeus Abiy, and

  • Mahindra Jain
  • Jenna Nieminen
  • Karthik Sharma
  • Pranshu Gaba
  • Tapas Mazumdar
  • Jimin Khim
  • Calvin Lin

contributed

Suppose there are continuous functions \(f(x)\) and \(g(x)\) that are both zero at \(x=a\). Then, the limit \({\displaystyle \lim_{x \to a}}{\frac{f(x)}{g(x)}}\) cannot be found by substituting \(x=a\), since it yields \(\frac00\) which cannot be evaluated. We use \(\frac00\) as a notation for an expression known as an indeterminate form. In some cases, limits that lead to indeterminate forms may be evaluated by cancellation or rearrangement of terms (see Limits by Factoring for examples). However, this does not always work. For example, how would you evaluate \({\displaystyle \lim_{x \to 0}}{\frac{\sin{x}}{x}}?\) Obviously, inserting \(x=0\) will yield an indeterminate form of \(\frac00\) and, in this case, you can neither use algebraic manipulation nor rearrangement of terms to reduce this expression into a form that yields a valid limit.

Under certain circ*mstances, we can use a powerful theorem called L'Hôpital's rule to evaluate the limits that lead to indeterminate forms.

Contents

  • L'Hopital Rule
  • Basic Examples
  • Intermediate Examples
  • Repeated Application of L'Hopital's Rule - Basic
  • Repeated Application of L'Hopital's Rule - Intermediate
  • Problem Solving
  • Convergence of Sequences
  • Convergence of Improper Integrals

L'Hopital Rule

L'Hopital's Rule

Suppose \(f\) and \(g\) are differentiable functions such that

  1. \(g'(x) \neq 0\) on an open interval \(I\) containing \(a;\)
  2. \(\displaystyle \lim_{x\to a} f(x) = 0 \text{ and } \displaystyle \lim_{x\to a} g(x) = 0,\) or \( \displaystyle \lim_{x\to a} f(x) = \pm \infty \text{ and }\displaystyle \lim_{x\to a} g(x) = \pm \infty; \)
  3. \({\displaystyle \lim_{x\to a}} \frac{f'(x)}{g'(x)} \) exists.

Then

\[\lim_{x\to a} \frac{f(x)}{g(x)} = \lim_{x\to a} \frac{f'(x)}{g'(x)}. \]

We have

\[\begin{align}\frac{f'(a)}{g'(a)} &= \frac{\lim_{x \to a}{\frac{f(x) - f(a)}{x - a}}}{\lim_{x \to a}{\frac{g(x) - g(a)}{x - a}}} \\\\&= \lim_{x \to a}{\frac{\hspace{4mm} \frac{f(x)-f(a)}{x-a}\hspace{4mm} }{\frac{g(x)-g(a)}{x-a}}} \\\\&= \lim_{x \to a}{\frac{f(x)-f(a)}{g(x)-g(a)}} \\\\&= \lim_{x \to a}{\frac{f(x) - 0}{g(x) - 0}} \\\\&= \lim_{x \to a}{\frac{f(x)}{g(x)}}. \ _\square\end{align}\]

Basic Examples

Evaluate

\[\lim_{x \to 0}{\frac{\sin{x}}{x}}.\]

Directly applying \(x=0\) leads the limit to an indeterminate form. Since both the term in the numerator and the term in the denominator are zero at \(x = 0\) and since \(\sin{x}\) and \(x\) are both differentiable at \(x=0\), we can use L'Hôpital's rule:

\[\lim_{x \to 0}{\frac{\sin{x}}{x}} = \left. \frac{\cos{x}}{1}\right|_{x = 0} = \frac{1}{1} = 1 .\ _\square\]

Evaluate

\[\lim_{x \to 0}{\frac{\sqrt{x+9}-3}{x}}.\]

Once again, directly applying \(x=0\) produces an indeterminate form. Let the expression in the numerator be \(f(x),\) and the expression in the denominator \(g(x)\). Since \(f(0)\) and \(g(0)\) are both zero and \(f(x)\) and \(g(x)\) are both differentiable at \(x=0\), we can use L'Hôpital's rule:

\[\lim_{x \to 0}{\frac{\sqrt{x+9}-3}{x}} = \left.\frac{\hspace{3mm} \frac{1}{2\sqrt{x+9}}\hspace{3mm} }{1}\right|_{x=0} = \frac{1}{6}.\ _\square\]

Calculate \(\displaystyle{ \lim_{x\to \infty } \frac{\ln(x)}{x^{^{\frac{1}{3}}}} }. \)

Since \(\ln(x) \to \infty\) and \(x^{^{\frac{1}{3}}} \to \infty \) as \(x \to \infty\), we can use L'Hopital's rule:

\[ \lim_{x\to \infty } \frac{\ln(x)}{x^{^{\frac{1}{3}}}} = \lim_{x\to \infty } \frac{\dfrac{1}{x}}{\frac{1}{3}x^{^{-\frac{2}{3}}}} . \]

Simplifying the expression, we obtain

\[ \lim_{x\to \infty } \frac{3}{x^{^\frac{1}{3}}} = 0. \ _\square \]

0 0.512 1 \[ \dfrac { \pi}{\pi - 1} \]

For a constant \(k\),

\[ \lim_{ n \to \infty } n^{n^{-1} } = e^k .\]

Find the value of \( \ln \big( k^2 +1 \big). \)

Intermediate Examples

So far we have looked at evaluating limits that reduce to the indeterminate form

\[\frac{0}{0}\quad \text{ or }\quad \frac{\infty}{\infty}.\]

But how would we evaluates limits that reduce to \(0\times\infty?\) We can deal with such forms by writing them as quotients or rearranging them. The following identity is helpful:

\[f \times g = \frac{f}{\hspace{2mm} \frac{1}{g}\hspace{2mm} } .\]

Evaluate \(\displaystyle{ \lim_{x\to 0^+ } x\ln(x)}. \)

We have

\[ \lim_{x\to 0^+}{ x\ln(x) } = \lim_{x\to 0^+} \frac{\ln(x)}{\frac{1}{x}} = \lim_{x\to 0^+} \frac{x^{-1}}{-x^{-2}} = 0. \ _\square \]

Evaluate

\[\lim_{x \to \infty}{x \sin{\frac{1}{x}}}.\]

Directly applying \(x=0\) gives an indeterminate form of \(\infty \cdot 0\). This form of limit cannot be evaluated just like the case of \(\frac00\). This seemingly formidable problem can be solved by introducing a variable substitution, \(x = \frac{1}{y}\). Using the substitution, the limit expression is changed as

\[\lim_{y \to 0^{+}}{\frac{1}{y} \sin{y}} = 1.\ _\square\]

If the limit

\[\lim _{ x\rightarrow 0 }{ \left( \frac { \sin { 2x } }{ { x }^{ 3 } } +a+\frac { b }{ { { x }^{ 2 } } } \right) } =0\]

is true for constants \(a\) and \(b\), then what is the value of \(3a+b?\)

Repeated Application of L'Hopital's Rule - Basic

In the case where application of L'Hôpital's rule yields an indeterminate form, if the resulting limit expression meets the conditions necessary to use L'Hôpital's rule, it can be used again. This can be quite confusing to understand. Let's look at the example below to see what this means.

Evaluate

\[\lim_{x \to 0}{\frac{x - \sin{x}}{x^3}}.\]

Let \(f(x)=x - \sin{x}\) and \(g(x)=x^3.\) Since \(f(0) = g(0) = 0\), and \(f(x)\) and \(g(x)\) are both differentiable at \(x=0\), we can use L'Hôpital's rule as follows:

\[\lim_{x \to 0}{\frac{x - \sin{x}}{x^3}} = \lim_{x \to 0}{\frac{1 - \cos{x}}{3x^2}} = \frac{0}{0}.\]

The resulting expression yields an indeterminate form of \(\frac00\). However, the term meets the criteria required to use L'Hôpital's rule:

\[\begin{align}\lim_{x \to 0}{\frac{1 - \cos{x}}{3x^2}} &= \lim_{x \to 0}{\frac{\sin{x}}{6x}} = \frac{0}{0}\\ \\& \Rightarrow \lim_{x \to 0}{\frac{\cos{x}}{6}} = \frac{1}{6}. \ _\square\end{align}\]

Evaluate

\[ \lim_{x\to0} \frac{\tan(123x)}{\tan(456x)}. \]

Because the limit is in the form of \( \frac00 \), we can apply L'Hôpital's rule. The derivative of \(\tan x \) is \(\sec^2 x \), and thus by the chain rule, \( \frac d{dx} \tan Ax = A \sec^2 Ax \) for some constant \(A\). Thus, the limit becomes

\[ \lim_{x\to0} \frac{123\sec^2(123x)}{456\sec^2(456x)} = \frac{123\sec^2(0)}{456\sec^2(0)} = \frac{41}{152}. \ _\square \]

Note: We can also solve this limit by using the approximation \( \tan x \approx x \) for relatively small \(x\).

Evaluate the limit

\[ \lim_{x\to0} \frac{\sin x}{\tan x}.\]

Note that the limit is indeed in the form of \( \frac00.\) However, we can simplify the given expression by noting that \( \tan x = \frac{\sin x}{\cos x} \). So by taking their ratio, we are left with \(\cos x, \) and its limit when \(x\) approaches \(0\) is simply \(1. \ _\square\)

What is the value of

\[ \lim _{ x \rightarrow 0 } \frac{ \tan x - x } { \sin x - x }? \]

Repeated Application of L'Hopital's Rule - Intermediate

Evaluate the limit

\[\lim_{x\to0} \frac{1-\cos x^2}{x^4}.\]

Since the limit has the form of \( \frac00 \) when \(x=0\), L'Hôpital's rule is applicable. Since \( \frac d{dx} (1- \cos x^2) = 2x \sin x^2,\) we have

\[\begin{eqnarray} \displaystyle \lim_{x\to0} \frac{1-\cos x^2}{x^4} \displaystyle &=& \lim_{x\to0} \frac{2x\sin x^2}{4x^3} \\\displaystyle &=& \lim_{x\to0} \frac24 \cdot \frac{\sin x^2}{x^2} \\\displaystyle &=& \lim_{x^2\to0} \frac12 \cdot \frac{\sin x^2}{x^2} \\\displaystyle &=& \frac12 \lim_{y\to0} \cdot \frac{\sin y}{y} \\\displaystyle &=& \frac12 \cdot 1 = \frac12. \ _\square \end{eqnarray} \]

Given that \(A,B\) and \(C\) are finite constants such that \( {\displaystyle \lim_{x\to0}} \frac{\sin x +Ax + Bx^3}{x^5} = \frac1C ,\) evaluate \(A\times B\times C\).

We first note that the limit is of the form \( \frac00\) when \(x=0\), so L'Hôpital's rule is applicable:

\[\begin{align}\lim_{x\to0} \frac{\sin x +Ax + Bx^3}{x^5} &=\lim_{x\to0} \frac{ \frac d{dx} \left[\sin x +Ax + Bx^3 \right] } { \frac d{dx} (x^5) } \\&=\lim_{x\to0} \frac{ \frac d{dx} \left[\cos x +A +3Bx^2 \right] } { 5x^4 }.\end{align}\]

If \(A \ne -1\), the limit equals to \( \frac A0 ,\) which cannot be equal to a finite constant \(C.\) Hence \(A \) is forced to take the value of \(-1 \). Continue by applying the rule a few more times to obtain

\[\begin{align}\lim_{x\to0} \frac{\sin x +Ax + Bx^3}{x^5}&=\lim_{x\to0} \frac{\cos x +A +3Bx^2 } { 5x^4 } \\&=\lim_{x\to0} \frac{-\sin x + 6Bx}{20x^3} \\&=\lim_{x\to0} \frac{-\cos x + 6B}{60x^2}.\end{align} \]

Similarly, as above, \(6B \) is forced to take a value of \(1,\) so \(B = \frac16 \). Then we have

\[\begin{align}\lim_{x\to0} \frac{\sin x +Ax + Bx^3}{x^5} &=\lim_{x\to0} \frac{-\cos x + 1}{60x^2} \\&=\lim_{x\to0} \frac{\sin x}{120x} \\&=\lim_{x\to0} \frac{\cos x}{120} \\&= \frac1{120}. \end{align} \]

Hence, \(\frac1C = \frac1{120} \) or \(C = 120,\) implying

\[A \times B \times C = -1 \times \frac16 \times 120 = -20. \ _\square\]

The limit

\[ \lim_{x \to 3} \left [ \frac{1}{(9-x^2)^{2}} \left( \frac{9+x^2}{3x} - 2\sin \frac{3π}{2} \sin \frac{πx}{2} \right) \right ] \]

can be expressed as \( \displaystyle \frac{a + b \pi^ c}{d^a} \) for positive integers \(a,b,c,\) and \(d.\)

What is the value of \(a+b+c+d?\)

Please help me! I kept using L'Hôpital's rule millions of times and I can't evaluate the limit below!

\[ \displaystyle \lim_{x \to 0} \frac {\cot x }{\csc x } = \, ? \]

Details and Assumptions:

  • \( \frac{d}{dx}( \cot x ) = -\csc^2 x \)
  • \( \frac{d}{dx}( \csc x ) = -\csc x \cot x \)

Problem Solving

Let \(f(x) = x e^2\) and \(g(x) = x^{\ln x},\) and let \(\alpha\) and \(\beta\) with \(\alpha < \beta\) be the two roots of \(f(x)-g(x) = 0.\) Also, let

\[ l= \lim_{x \rightarrow \beta} \ \dfrac{f(x)-c\beta}{g(x)-\beta^2}. \]

Then what is the value of \(c-l?\)

Note: You may use the approximation \( e \approx 2.7183 .\)

Try my set.

The equation \( \displaystyle \lim_{x \to 0} \int_0^x \frac {t^2 \mathrm d t}{(x- \sin x) \sqrt{a+t} } = 1 \) is true for a constant \(a\).

What is the value of \(a?\)

\[ \lim_{x \to 0} \left(\frac{1}{x^4} - \frac{ {\displaystyle\int_{0}^{x^2}} e^{-u^2} du}{x^6}\right) = \, ? \]

Convergence of Sequences

Convergence of Improper Integrals

Cite as: L'Hôpital's Rule. Brilliant.org. Retrieved from https://brilliant.org/wiki/lhopitals-rule/

L'Hôpital's Rule | Brilliant Math & Science Wiki (2024)

FAQs

What is the L Hopital rule? ›

The tangent to the curve at the point [g(t), f(t)] is given by [g′(t), f′(t)]. L'Hôpital's rule then states that the slope of the curve when t = c is the limit of the slope of the tangent to the curve as the curve approaches the origin, provided that this is defined.

Does l'hôpital's rule always work? ›

When can you not use the L'Hospital's rule? L'Hospital's Rule only applies when the expression is indeterminate, i.e. 0/0 or (+/-infinity)/(+/-infinity). Hence, we have to stop applying the rule when you have a deductive form.

What is the L hospital rule in real analysis? ›

L' Hospital's rule states that, when the limit of f(x)g(x) is indeterminate, under certain conditions it can be obtained by evaluating the limit of the quotient of the derivatives of f and g (i.e., f′(x)g′(x)). If this result is indeterminate, the procedure can be repeated.

What is the significance of L Hospital rule? ›

Overview and Proof of L'Hospital's Rule. This is a rule that helps to evaluate the limits which involve indeterminate forms by using the derivatives. An indeterminate form can be defined as a limit that does not provide enough information to determine the original limit. It is a very important rule in Calculus.

When should I use L Hopital? ›

When you are solving a limit, and get 0/0 or ∞/∞, L'Hôpital's rule is the tool you need.

What is the L Hopital rule a level? ›

L'Hôpital's Rule

It says that the limit when we divide one function by another is the same after we take the derivative of each function (with some special conditions shown later).

What is the L hospital rule first principle? ›

L Hospital rule states that if the limit x--->a of f(x)/g(x) is of the form 0/0 or infinity/infinity, then the limit is equal to f'(x)/g'(x), where f'(x) represents the first derivative of f(x) wrt x.

What is the abstract of L Hospital rule? ›

Formally, L'Hopital's rule says that if you have some functions, like f(x) and g(x), and both of them approach zero as x goes to some number, like C, then the limit as x approaches C of the ratio of these functions is equal to the limit as x approaches C of the ratio of the derivatives of these functions.

How is L Hopital's rule used in real life? ›

Continuous compounding interest rates encountered everyday in investments, different types of bank accounts, or when paying credit cards bills, mortgages, etc. In short, L'Hospital's rule has many applications in the real world, predominantly in statistics, physics, and engineering!

What is the meaning of L Hopital? ›

L'hôpital means "The Hospital" in French.

Can you use L'Hôpital's rule for infinity over infinity? ›

Note that both x and e^x approach infinity as x approaches infinity, so we can use l'Hôpital's Rule. Also, the derivative of x is 1, and the derivative of e^x is (still) e^x.

What is the L hospital rule in complex analysis? ›

L'Hôpital's rule says that the limit as x goes to C of f(x)/g(x) is equal to the limit as x goes to C of f'(x)/g'(x) as long as your original limit gave to 0/0 or infinity/infinity.

What is the L hospital chain rule? ›

If f(x) and g(x) are continuous functions with either f(a)=g(a)=0, or f(a)=±∞ and g(a)=±∞, then limx→af(x)g(x)=limx→af′(x)g′(x). Warning: While L'Hospital's Rule says that the limits of fg and f′g′ are equal under certain circ*mstances, it is not true that fg=f′g′!

References

Top Articles
Latest Posts
Article information

Author: Amb. Frankie Simonis

Last Updated:

Views: 6228

Rating: 4.6 / 5 (76 voted)

Reviews: 91% of readers found this page helpful

Author information

Name: Amb. Frankie Simonis

Birthday: 1998-02-19

Address: 64841 Delmar Isle, North Wiley, OR 74073

Phone: +17844167847676

Job: Forward IT Agent

Hobby: LARPing, Kitesurfing, Sewing, Digital arts, Sand art, Gardening, Dance

Introduction: My name is Amb. Frankie Simonis, I am a hilarious, enchanting, energetic, cooperative, innocent, cute, joyous person who loves writing and wants to share my knowledge and understanding with you.